You are on page 1of 60

Emergency Nursing Quiz - I

This page was last updated on April 5, 2011


1. Which of the follwoing is the most effective benzodiazepine for treating status
epilepticus and is the treatment of choice for controlling seizures acutely?

A. Midazolam

B. Oxazepam

C. Flurazepam

D. Lorazepam

Answer Key
2. Subdiaphragmatic abdominal thrust performed to clear airway obstruction in the
awake patient without ventilation is termed as:

A. Leopold's maneuvers

B. Pallach's maneuver

C. Heimlich maneuver

D. Valsalva maneuver

Answer Key
3. A slight bluish discoloration around the navel, is a sign of hemoperitoneum,

A. Cullen's sign

B. Kernig’s sign

C. Brudzinski’s signs

D. Kehr's sign

Answer Key
4. Signs of meningeal irritation include all the following, EXCEPT:

A. Neck stiffness

B. Brudzinski's sign

C. Kernig’s sign

D. Kehr's sign

Answer Key
5. The APACHE II system is a:

A. Quality assurance system


B. Severity-of-illness scoring system

C. Computerized vital signsmonitoring system

D. Neborn hydration monitoring system

Answer Key
6. A client is admitted to the emergency department following an automobile accident. The
client has four fractured ribs and a right sided pneumothorax. Which of the following
respiratory assessment findings would the nurse expect to find?

A. Crackles on the right chest and a respiratory rate of 8 breaths/minute.

B. Diminished breath sounds on the right and pain on inspiration.

C. Bilateral rhonchi and pink frothy sputum.

D. Dry cough and wheezing on the right side of the chest.

Answer Key
7. Clinical evidence of diminished Cardiac Output includes:

A. a narrow pulse pressure

B. rapid capillary refill

C. widened pulse pressure

D. bounding pulses

Answer Key
8. The most common cause of high cardiac output hypotension is:

A. hemorrhage

B. cardiac failure

C. sepsis

D. neurogenic shock

Answer Key
9. Intermediate Syndrome is observed in:

A. cardiac arrest

B. respiratory failure

C. renal failure

D. organophosphorous poisoning

Answer Key
10. The procedure of compressing cricothyroid cartilage posteriorly against vertebral
bodies during endotracheal intubation is called:

A. Valsalva maneuver

B. Sellick maneuver

C. Heimlich maneuver

D. Leopold's maneuvers

Answer Key
11. Which of the following pairing about Glasgow Coma Score is wrong?

A. 6 —Spontaneous Eye Opening

B. 1—No Verbal Response

C. 5—Localizes to pain (Motor Activity)

D. 3—Eye Opening to Verbal stimuli

Answer Key

Answer Key

1.D 2. C 3. A 4. D 5. B
6. A 7. A 8. C 9. D 10. B
11. A BACK TO TOP
Back to Quiz Corner

1. The nurse is triaging four clients injured in a train derailment. Which client should
receive priority treatment?
A. A 42-year-old with dyspnea and chest asymmetry
B. A 17-year-old with a fractured arm
C. A 4-year-old with facial lacerations
D. A 30-year-old with blunt abdominal trauma

2. Direct pressure to a deep laceration on the client’s lower leg has failed to stop the
bleeding. The nurse’s next action should be to:
A. Place a tourniquet proximal to the laceration.
B. Elevate the leg above the level of the heart.
C. Cover the laceration and apply an ice compress.
D. Apply pressure to the femoral artery.

3. A pediatric client is admitted after ingesting a bottle of vitamins with iron.


Emergency care would include treatment with:
A. Acetylcysteine
B. Deferoxamine
C. Calcium disodium acetate
D. British antilewisite

4. The nurse is preparing to administer Ringer’s Lactate to a client with hypovolemic


shock. Which intervention is important in helping to stabilize the client’s condition?
A. Warming the intravenous fluids
B. Determining whether the client can take oral fluids
C. Checking for the strength of pedal pulses
D. Obtaining the specific gravity of the urine

5. The emergency room staff is practicing for its annual disaster drill. According to
disaster triage, which of the following four clients would be cared for last?
A. A client with a pneumothorax
B. A client with 70% TBSA full thickness burns
C. A client with fractures of the tibia and fibula
D. A client with smoke inhalation injuries

6. An unresponsive client is admitted to the emergency room with a history of


diabetes mellitus. The client’s skin is cold and clammy, and the blood pressure
reading is 82/56. The first step in emergency treatment of the client’s symptoms
would be:
A. Checking the client’s blood sugar
B. Administering intravenous dextrose
C. Intubation and ventilator support
D. Administering regular insulin

7. A client with a history of severe depression has been brought to the emergency
room with an overdose of barbiturates. The nurse should pay careful attention to
the client’s:
A. Urinary output
B. Respirations
C. Temperature
D. Verbal responsiveness
8. A client is to receive antivenin following a snake bite. Before administering the
antivenin, the nurse should give priority to:
A. Administering a local anesthetic
B. Checking for an allergic response
C. Administering an anxiolytic
D. Withholding fluids for 6–8 hours

9. The nurse is caring for a client following a radiation accident. The client is
determined to have incorporation. The nurse knows that the client will:
A. Not need any medical treatment for radiation exposure
B. Have damage to the bones, kidneys, liver, and thyroid
C. Experience only erythema and desquamation
D. Not be radioactive because the radiation passes through the body

10. The emergency staff has undergone intensive training in the care of clients with
suspected anthrax. The staff understands that the suggested drug for treating
anthrax is:
A. Ancef (cefazolin sodium)
B. Cipro (ciprofloxacin)
C. Kantrex (kanamycin)
D. Garamycin (gentamicin)

Answer Rationales

1. Answer A is correct. Following the ABCDs of basic emergency care, the client with
dyspnea and asymmetrical chest should be cared for first because these
symptoms are associated with flail chest. Answer D is incorrect because he should
be cared for second because of the likelihood of organ damage and bleeding.
Answer B is incorrect because he should be cared for after the client with
abdominal trauma. Answer C is incorrect because he should receive care last
because his injuries are less severe.

2. Answer B is correct. If bleeding does not subside with direct pressure, the nurse
should elevate the extremity above the level of the heart. Answers A and D are
done only if other measures are ineffective, so they are incorrect. Answer C would
slow the bleeding but will not stop it, so it’s incorrect.

3. Answer B is correct. Deferoxamine is the antidote for iron poisoning. Answer A is


the antidote for acetaminophen overdose, making it wrong. Answers C and D are
antidotes for lead poisoning, so they are wrong.

4. Answer A is correct. Warming the intravenous fluid helps to prevent further stress
on the vascular system. Thirst is a sign of hypovolemia; however, oral fluids alone
will not meet the fluid needs of the client in hypovolemic shock, so answer B is
incorrect. Answers C and D are wrong because they can be used for baseline
information but will not help stabilize the client.

5. Answer B is correct. The client with 70% TBSA burns would be classified as an
emergent client. In disaster triage, emergent clients, code black, are cared for last
because they require the greatest expenditure of resources. Answers A and D are
examples of immediate clients and are assigned as code red, so they are wrong.
These clients are cared for first because they can survive with limited
interventions. Answer C is wrong because it is an example of a delayed client,
code yellow. These clients have significant injuries that require medical care.

6. Answer A is correct. The client has symptoms of insulin shock and the first step is
to check the client’s blood sugar. If indicated, the client should be treated with
intravenous dextrose. Answer B is wrong because it is not the first step the nurse
should take. Answer C is wrong because it does not apply to the client’s
symptoms. Answer D is wrong because it would be used for diabetic ketoacidosis,
not insulin shock.

7. Answer B is correct. Barbiturate overdose results in central nervous system


depression, which leads to respiratory failure. Answers A and C are important to
the client’s overall condition but are not specific to the question, so they are
incorrect. The use of barbiturates results in slow, slurred speech, so answer D is
expected, and therefore incorrect.

8. Answer B is correct. The nurse should perform the skin or eye test before
administering antivenin. Answers A and D are unnecessary and therefore
incorrect. Answer C would help calm the client but is not a priority before giving
the antivenin, making it incorrect.

9. Answer B is correct. The client with incorporation radiation injuries requires


immediate medical treatment. Most of the damage occurs to the bones, kidneys,
liver, and thyroid. Answers A, C, and D refer to external irradiation, so they are
wrong.

10. Answer B is correct. Cipro (ciprofloxacin) is the drug of choice for treating anthrax.
Answers A, C, and D are not used to treat anthrax, so they are incorrect.

1. You are the charge nurse in an emergency department (ED) and must assign two staff members to
cover the triage area. Which team is the most appropriate for this assignment?

a. An advanced practice nurse and an experienced LPN/LVN

b. An experienced LPN/LVN and an inexperienced RN

c. An experienced RN and an inexperienced RN

d. An experienced RN and a nursing assistant


2. You are working in the triage area of an ED, and four patients approach the triage desk at the same
time. List the order in which you will assess these patients.

a. An ambulatory, dazed 25-year-old male with a bandaged head wound

b. An irritable infant with a fever, petechiae, and nuchal rigidity

c. A 35-year-old jogger with a twisted ankle, having pedal pulse and no deformity

d. A 50-year-old female with moderate abdominal pain and occasional vomiting

3. In conducting a primary survey on a trauma patient, which of the following is considered one of the
priority elements of the primary survey?

a. Complete set of vital signs

b. Palpation and auscultation of the abdomen

c. Brief neurologic assessment

d. Initiation of pulse oximetry

4. A 56-year-old patient presents in triage with left-sided chest pain, diaphoresis, and dizziness. This
patient should be prioritized into which category?

a. High urgent

b. Urgent

c. Non-urgent

d. Emergent

5. The physician has ordered cooling measures for a child with fever who is likely to be discharged when
the temperature comes down. Which of the following would be appropriate to delegate to the nursing
assistant?

a. Assist the child to remove outer clothing.

b. Advise the parent to use acetaminophen instead of aspirin.

c. Explain the need for cool fluids.

d. Prepare and administer a tepid bath.

6. It is the summer season, and patients with signs and symptoms of heat-related illness present in the
ED. Which patient needs attention first?
a. An elderly person complains of dizziness and syncope after standing in the sun for several hours to
view a parade

b. A marathon runner complains of severe leg cramps and nausea. Tachycardia, diaphoresis, pallor, and
weakness are observed.

c. A previously healthy homemaker reports broken air conditioner for days. Tachypnea, hypotension,
fatigue, and profuse diaphoresis are observed.

d. A homeless person, poor historian, presents with altered mental status, poor muscle coordination,
and hot, dry, ashen skin. Duration of exposure is unknown.

7. You respond to a call for help from the ED waiting room. There is an elderly patient lying on the floor.
List the order for the actions that you must perform.

a. Perform the chin lift or jaw thrust maneuver.

b. Establish unresponsiveness.

c. Initiate cardiopulmonary resuscitation (CPR).

d. Call for help and activate the code team. e. Instruct a nursing assistant to get the crash cart.

8. The emergency medical service (EMS) has transported a patient with severe chest pain. As the patient
is being transferred to the emergency stretcher, you note unresponsiveness, cessation of breathing, and
no palpable pulse. Which task is appropriate to delegate to the nursing assistant?

a. Chest compressions

b. Bag-valve mask ventilation

c. Assisting with oral intubation

d. Placing the defibrillator pads

9. An anxious 24-year-old college student complains of tingling sensations, palpitations, and chest
tightness. Deep, rapid breathing and carpal spasms are noted. What priority nursing action should you
take?

a. Notify the physician immediately.

b. Administer supplemental oxygen.

c. Have the student breathe into a paper bag.

d. Obtain an order for an anxiolytic medication.


10.An experienced traveling nurse has been assigned to work in the ED; however, this is the nurse’s first
week on the job. Which area of the ED is the most appropriate assignment for the nurse?

a. Trauma team

b. Triage

c. Ambulatory or fats track clinic

d. Pediatric medicine team

11.A tearful parent brings a child to the ED for taking an unknown amount of children’s chewable
vitamins at an unknown time. The child is currently alert and asymptomatic. What information should
be immediately reported to the physician?

a. The ingested children’s chewable vitamins contain iron.

b. The child has been treated several times for ingestion of toxic substances.

c. The child has been treated several times for accidental injuries.

d. The child was nauseated and vomited once at home.

12.In caring for a victim of sexual assault, which task is most appropriate for an LPN/LVN?

a. Assess immediate emotional state and physical injuries

b. Collect hair samples, saliva swabs, and scrapings beneath fingernails.

c. Provide emotional support and supportive communication.

d. Ensure that the “chain of custody” is maintained.

13.You are caring for a victim of frostbite to the feet. Place the following interventions in the correct
order.

a. Apply a loose, sterile, bulky dressing.

b. Give pain medication.

c. Remove the victim from the cold environment.

d. Immerse the feet in warm water 100o F to 105o F (40.6o C to 46.1o C)

14.A patient sustains an amputation of the first and second digits in a chainsaw accident. Which task
should be delegated to the LPN/LVN?

a. Gently cleanse the amputated digits with Betadine solution.


b. Place the amputated digits directly into ice slurry.

c. Wrap the amputated digits in sterile gauze moistened with saline.

d. Store the amputated digits in a solution of sterile normal saline.

15.A 36-year-old patient with a history of seizures and medication compliance of phenytoin (Dilantin)
and carbamazepine (Tegretol) is brought to the ED by the MS personnel for repetitive seizure activity
that started 45 minutes prior to arrival. You anticipate that the physician will order which drug for status
epilepticus?

a. PO phenytoin and carbamazepine

b. IV lorazepam (Ativan)

c. IV carbamazepam

d. IV magnesium sulfate

16.You are preparing a child for IV conscious sedation prior to repair of a facial laceration. What
information should you immediately report to the physician?

a. The parent is unsure about the child’s tetanus immunization status.

b. The child is upset and pulls out the IV.

c. The parent declines the IV conscious sedation.

d. The parent wants information about the IV conscious sedation.

17.An intoxicated patient presents with slurred speech, mild confusion, and uncooperative behavior.
The patient is a poor historian but admits to “drinking a few on the weekend.” What is the priority
nursing action for this patient?

a. Obtain an order for a blood alcohol level.

b. Contact the family to obtain additional history and baseline information.

c. Administer naloxone (Narcan) 2 – 4 mg as ordered.

d. Administer IV fluid support with supplemental thiamine as ordered.

18.When an unexpected death occurs in the ED, which of the following tasks is most appropriate to
delegate to the nursing assistant?

a. Escort the family to a place of privacy.

b. Go with the organ donor specialist to talk to the family.


c. Assist with postmortem care.

d. Assist the family to collect belongings.

19.Following emergency endotracheal intubation, you must verify tube placement and secure the tube.
List in order the steps that are required to perform this function?

a. Obtain an order for a chest x-ray to document tube placement.

b. Secure the tube in place.

c. Auscultate the chest during assisted ventilation.

d. Confirm that the breath sounds are equal and bilateral. _____, _____, _____, _____

20.A teenager arrives by private car. He is alert and ambulatory, but this shirt and pants are covered
with blood. He and his hysterical friends are yelling and trying to explain that that they were goofing
around and he got poked in the abdomen with a stick. Which of the following comments should be
given first consideration?

a. “There was a lot of blood and we used three bandages.”

b. “He pulled the stick out, just now, because it was hurting him.”

c. “The stick was really dirty and covered with mud.”

d. “He’s a diabetic, so he needs attention right away.”

21.A prisoner, with a known history of alcohol abuse, has been in police custody for 48 hours. Initially,
anxiety, sweating, and tremors were noted. Now, disorientation, hallucination, and hyper-reactivity are
observed. The medical diagnosis is delirium tremens. What is the priority nursing diagnosis?

a. Risk for Injury related to seizures

b. Risk for Other-Directed Violence related to hallucinations

c. Risk for Situational Low Self-esteem related to police custody

d. Risk for Nutritional Deficit related to chronic alcohol abuse

22.You are assigned to telephone triage. A patient who was stung by a common honey bee calls for
advice, reports pain and localized swelling, but denies any respiratory distress or other systemic signs of
anaphylaxis. What is the action that you should direct the caller to perform?

a. Call 911.

b. Remove the stinger by scraping.

c. Apply a cool compress.


d. Take an oral antihistamine.

23.In relation to submersion injuries, which task is most appropriate to delegate to an LPN/LVN?

a. Talk to a community group about water safety issues.

b. Stabilize the cervical spine for an unconscious drowning victim.

c. Remove wet clothing and cover the victim with a warm blanket.

d. Monitor an asymptomatic near-drowning victim.

24.You are assessing a patient who has sustained a cat bite to the left hand. The cat is up-to-date
immunizations. The date of the patient’s last tetanus shot is unknown. Which of the following is the
priority nursing diagnosis?

a. Risk for Infection related to organisms specific to cat bites

b. Impaired Skin Integrity related to puncture wounds

c. Ineffective Health Maintenance related to immunization status

d. Risk for Impaired Mobility related to potential tendon damage

25.These patients present to the ED complaining of acute abdominal pain. Prioritize them in order of
severity.

a. A 35-year-old male complaining of severe, intermittent cramps with three episodes of watery
diarrhea, 2 hours after eating

b. A 11-year-old boy with a low-grade fever, left lower quadrant tenderness, nausea, and anorexia for
the past 2 days

c. A 40-year-old female with moderate left upper quadrant pain, vomiting small amounts of yellow bile,
and worsening symptoms over the past week

d. A 56-year-old male with a pulsating abdominal mass and sudden onset of pressure-like pain in the
abdomen and flank within the past hour _____, _____, _____, _____

26.The nursing manager decides to form a committee to address the issue of violence against ED
personnel. Which combination of employees is best suited to fulfill this assignment?

a. ED physicians and charge nurses

b. Experienced RNs and experienced paramedics

c. RNs, LPN/LVNs, and nursing assistants

d. At least one representative from each group of ED personnel


27.In a multiple-trauma victim, which assessment finding signals the most serious and life-threatening
condition?

a. A deviated trachea

b. Gross deformity in a lower extremity

c. Decreased bowel sounds

d. Hematuria

28.A patient in a one-car rollover presents with multiple injuries. Prioritize the interventions that must
be initiated for this patient.

a. Secure/start two large-bore IVs with normal saline

b. Use the chin lift or jaw thrust method to open the airway.

c. Assess for spontaneous respirations

d. Give supplemental oxygen per mask.

e. Obtain a full set of vital signs.

f. Remove patient’s clothing.

g. Insert a Foley catheter if not contraindicated. _____, _____, _____, _____, ____, ____, ____

29.In the work setting, what is your primary responsibility in preparing for disaster management that
includes natural disasters or bioterrorism incidents?

a. Knowledge of the agency’s emergency response plan

b. Awareness of the signs and symptoms for potential agnets of bioterrorism

c. Knowledge of how and what to report to the CDC

d. Ethical decision-making about exposing self to potentially lethal substances

30.You are giving discharge instructions to a woman who has been treated for contusions and bruises
sustained during an episode of domestic violence. What is your priority intervention for this patient?

a. Transportation arrangements to a safe house

b. Referral to a counselor

c. Advise about contacting the police


d. Follow-up appointment for injuries

------------------------------------------------------------------------------------

1. Answer: C – Triage requires at least one experienced RN. Pairing an experienced RN with
inexperienced RN provides opportunities for mentoring. Advanced practice nurses are qualified to
perform triage; however, their services are usually required in other areas of the ED. An LPN/LVN is not
qualified to perform the initial patient assessment or decision making. Pairing an experienced RN with a
nursing assistant is the second best option, because the assistant can obtain vital signs and assist in
transporting.

2. Answer: B, A, D, C – An irritable infant with fever and petechiae should be further assessed for other
meningeal signs. The patient with the head wound needs additional history and assessment for
intracranial pressure. The patient with moderate abdominal pain is uncomfortable, but not unstable at
this point. For the ankle injury, medical evaluation can be delayed 24 – 48 hours if necessary.

3. Answer: C – A brief neurologic assessment to determine level of consciousness and pupil reaction is
part of the primary survey. Vital signs, assessment of the abdomen, and initiation of pulse oximetry are
considered part of the secondary survey.

4. Answer: D – Chest pain is considered an emergent priority, which is defined as potentially life-
threatening. Patients with urgent priority need treatment within 2 hours of triage (e.g. kidney stones).
Non-urgent conditions can wait for hours or even days. (High urgent is not commonly used; however, in
5-tier triage systems, High urgent patients fall between emergent and urgent in terms of the time
lapsing prior to treatment).

5. Answer: A – The nursing assistant can assist with the removal of the outer clothing, which allows the
heat to dissipate from the child’s skin. Advising and explaining are teaching functions that are the
responsibility of the RN. Tepid baths are not usually performed because of potential for rebound and
shivering.

6. Answer: D – The homeless person has symptoms of heat stroke, a medical emergency, which
increases risk for brain damage. Elderly patients are at risk for heat syncope and should be educated to
rest in cool area and avoid future similar situations. The runner is having heat crams, which can be
managed with rest and fluids. The housewife is experiencing heat exhaustion, and management includes
fluids (IV or parenteral) and cooling measures. The prognosis for recovery is good.

7. Answer: B, D, A, C, E – Establish unresponsiveness first. (The patient may have fallen and sustained a
minor injury.) If the patient is unresponsive, get help and have someone initiate the code. Performing
the chin lift or jaw thrust maneuver opens the airway. The nurse is then responsible for starting CPR.
CPR should not be interrupted until the patient recovers or it is determined that heroic efforts have
been exhausted. A crash cart should be at the site when the code team arrives; however, basic CPR can
be effectively performed until the team arrives.
8. Answer: A – Nursing assistants are trained in basic cardiac life support and can perform chest
compressions. The use of the bag-valve mask requires practice and usually a respiratory therapist will
perform this function. The nurse or the respiratory therapist should provide PRN assistance during
intubation. The defibrillator pads are clearly marked; however, placement should be done by the RN or
physician because of the potential for skin damage and electrical arcing.

9. Answer: C – The patient is hyperventilating secondary to anxiety, and breathing into a paper bag will
allow rebreathing of carbon dioxide. Also, encouraging slow breathing will help. Other treatments such
as oxygen and medication may be needed if other causes are identified.

10. Answer: C – The fast track clinic will deal with relatively stable patients. Triage, trauma, and
pediatric medicine should be staffed with experienced nurses who know the hospital routines and
policies and can rapidly locate equipment.

11. Answer: A – Iron is a toxic substance that can lead to massive hemorrhage, coma, shock, and hepatic
failure. Deferoxame is an antidote that can be used for severe cases of iron poisoning. Other information
needs additional investigation, but will not change the immediate diagnostic testing or treatment plan.

12. Answer: C – The LPN/LVN is able to listen and provide emotional support for her patients. The other
tasks are the responsibility of an RN or, if available, a SANE (sexual assault nurse examiner) who has
received training to assess, collect and safeguard evidence, and care for these victims.

13. Answer: C, B, D, A – The victim should be removed from the cold environment first, and then the
rewarming process can be initiated. It will be painful, so give pain medication prior to immersing the
feet in warmed water.

14. Answer: C – The only correct intervention is C. the digits should be gently cleansed with normal
saline, wrapped in sterile gauze moistened with saline, and placed in a plastic bag or container. The
container is then placed on ice.

15. Answer: B – IV Lorazepam (Ativan) is the drug of choice for status epilepticus. Tegretol is used in the
management of generalized tonic-clonic, absence or mixed type seizures, but it does not come in an IV
form. PO (per os) medications are inappropriate for this emergency situation. Magnesium sulfate is
given to control seizures in toxemia of pregnancy. Medical-Surgical Nursing

16. Answer: C – Parent refusal is an absolute contraindication; therefore, the physician must be notified.
Tetanus status can be addressed later. The RN can restart the IV and provide information about
conscious sedation; if the parent still notsatisfied, the physician can give more information.

17. Answer: D – The patient presents with symptoms of alcohol abuse and there is a risk for Wernicke’s
syndrome, which is caused by a thiamine deficiency. Multiples drug abuse is not uncommon; however,
there is nothing in the question that suggests an opiate overdose that requires naloxone. Additional
information or the results of the blood alcohol level are part of the total treatment plan but should not
delay the immediate treatment.
18. Answer: C – Postmortem care requires some turning, cleaning, lifting, etc., and the nursing assistant
is able to assist with these duties. The RN should take responsibility for the other tasks to help the family
begin the grieving process. In cases of questionable death, belongings may be retained for evidence, so
the chain of custody would have to be maintained.

19. Answer: C, D, B, A – Auscultating and confirming equal bilateral breath sounds should be performed
in rapid succession. If the sounds are not equal or if the sounds are heard over the mid-epigastric area,
tube placement must be corrected immediately. Securing the tube is appropriate while waiting for the x-
ray study.

20. Answer: B – An impaled object may be providing a tamponade effect, and removal can precipitate
sudden hemodynamic decompensation. Additional history including a more definitive description of the
blood loss, depth of penetration, and medical history should be obtained. Other information, such as the
dirt on the stick or history of diabetes, is important in the overall treatment plan, but can be addressed
later.

21. Answer: A – The patient demonstrates neurologic hyperactivity and is on the verge of a seizure.
Patient safety is the priority. The patient needs chlordiazepoxide (Librium) to decrease neurologic
irritability and phenytoin (Dilantin) for seizures. Thiamine and haloperidol (Haldol) will also be ordered
to address the other problems. The other diagnoses are pertinent but not as immediate.

22. Answer: B – The stinger will continue to release venom into the skin, so prompt removal of the
stinger is advised. Cool compresses and antihistamines can follow. The caller should be further advised
about symptoms that require 911 assistance.

23. Answer: D – The asymptomatic patient is currently stable but should be observed for delayed
pulmonary edema, cerebral edema, or pneumonia. Teaching and care of critical patients is an RN
responsibility. Removing clothing can be delegated to a nursing assistant.

24. Answer: A – Cat’s mouths contain a virulent organism, Pasteurella multocida, that can lead to septic
arthritis or bacteremia. There is also a risk for tendon damage due to deep puncture wounds. These
wounds are usually not sutured. A tetanus shot can be given before discharge.

25. Answer: D, B, C, A – The patient with a pulsating mass has an abdominal aneurysm that may rupture
and he may decompensate suddenly. The 11-year-old boy needs evaluation to rule out appendicitis. The
woman needs evaluation for gallbladder problems that appear to be worsening. The 35-year-old man
has food poisoning, which is usually self-limiting.

26. Answer: D – At least one representative from each group should be included because all employees
are potential targets fro violence in the ED.

27. Answer: A – A deviated trachea is a symptoms of tension pneumothorax. All of the other symptoms
need to be addressed, but are of lesser priority.
28. Answer: C, B, D, A, E, F, G – For a multiple trauma victim, many interventions will occur
simultaneously as team members assist in the resuscitation. Methods to open the airway such as the
chin lift or jaw thrust can be used simultaneously while assessing for spontaneous respirations.
However, airway and oxygenation are priority. Starting IVs for fluid resuscitation is part of supporting
circulation. (EMS will usually establish at least one IV in the field.) Nursing assistants can be directed to
take vitals and remove clothing. Foley catheter is necessary to closely monitor output.

29. Answer: A – In preparing for disasters, the RN should be aware of the emergency response plan. The
plan gives guidance that includes roles of team members, responsibilities, and mechanisms of reporting.
Signs and symptoms of many agents will mimic common complaints, such as flu-like symptoms.
Discussions with colleagues and supervisors may help the individual nurse to sort through ethical
dilemmas related to potential danger to self.

30. Answer: A – Safety is a priority for this patient, and she should not return to a place where violence
could reoccur. The other options are important for the long term management of this care.

Read more at Nurseslabs.com Medical-Surgical Nursing Exam 15: Emergency Nursing (30
Items) http://nurseslabs.com/medical-surgical-nursing-exam-15-emergency-nursing-30-items/2/

1. A patient arrives at the emergency department complaining of mid-sternal chest pain.


Which of the following nursing action should take priority?

A. A complete history with emphasis on preceding events.


B. An electrocardiogram.
C. Careful assessment of vital signs.
D. Chest exam with auscultation.

1. A nursing student is studying about disasters and emergency preparedness.


Which of the following statements by the nursing student depicts a correct
understanding of the difference between a disaster and an emergency?

Your Answer: “An emergency is an unforeseen combination of


circumstances calling for immediate action for a range of
victims.”

Rationale: # 1 is incorrect because disasters may be natural or man-made. #


3 is incorrect because man-made disasters are either accidental or
intentional. # 4 is incorrect because natural disasters, not emergencies, are
caused by acts of nature or emerging diseases.

Nursing Process: Assessment

Client Need: Safe, Effective Care Environment

Cognitive Level: Application

Objective: Distinguish the difference between an emergency and a disaster.


Strategy: Look at each statement to see if it clearly defines either a disaster
or emergency. Then select the correct answer.

2. An emergency room nurse is working when there is a bioterrorism attack in


the city. Which of the following statements is a correct with regard to injuries
or symptoms associated with a bioterrorism attack?

Your Answer: It is not uncommon for the results of a biological attack to be


made known several hours or days after the attack.

Rationale: Biological terrorism is the use of etiological agents (disease) to


cause harm or kill a population, food, and/or livestock. # 1 is incorrect
because the main purpose of biological weapon use is mass devastation. # 3
is incorrect because a biological attack may not be known for several hours or
days after the attack. # 4 is incorrect because detection is difficult as clients
go to a number of different health care facilities for treatment.

Nursing Process: Planning

Client Need: Safe, Effective Care Environment

Cognitive Level: Analysis

Objective: Describe the types of injuries or symptoms that are associated


with biological, chemical, or radiological terrorism.

Strategy: Examine each statement to look for a correct statement about


bioterrorism.

3. The nurse is caring for a client with a blast injury. Which of the following
nursing assessments would be most appropriate for this client?

Your Answer: Assess for vasovagal hypotension

Rationale: Blast injuries are the result of explosive munitions, often involving
car or package bombs. Care for persons injured by blast injuries typically
focuses on abdominal and lung injuries, penetrating wounds, traumatic
amputations, and burns. # 2 is incorrect because a concussion, closed and
open brain injury, stroke, spinal cord injury, and an air embolism-induced
injury could result from a blast injury. #3 is incorrect because asphyxia could
result from a hurricane injury. # 4 is incorrect because the client would have
hypovolemia as a result from a blast injury.

Nursing Process: Assessment

Client Need: Safe, Effective Care Environment

Cognitive Level: Analysis


Objective: Evaluate nursing interventions related to the treatment of injuries
related to biological, chemical, or radiological terrorism.

Strategy: Differentiate between the different types of common injuries and


the associated assessments necessary to care for the client.

4. An emergency room nurse is working when an Amtrak train derails. The


emergency room nurse knows that reverse triage may need to be instituted.
What is the rationale for using reverse triage?

Your Answer: Reverse triage works on the principle of the greatest good for
the greatest number.

Rationale: During a disaster, nurses may be expected to perform triage.


Triage means sorting. # 1 is incorrect because a mass casualty is an event
with more than 100 victims. # 2 is incorrect because it describes basic triage
and not reverse triage. # 3 is incorrect because victims least likely to survive
or are already dead are color-coded as black.

Nursing Process: Planning

Client Need: Safe, Effective Care Environment

Cognitive Level: Application

Objective: Explain the rationale for reverse triage in disasters versus


conventional triage in emergencies.

Strategy: Read each answer choice to decide which statement correctly


depicts the concepts of reverse and conventional triage.

5. There has been a radioactive explosion nearby. The emergency room nurse
must triage and manage the decontamination of the clients systematically.
Which of the following clients would be decontaminated first?

Your Answer: A client with the least injuries.

Rationale: Reverse triage works on the principle of the greatest good for the
greatest number. In this case, those persons who are the most ambulatory
and least injured would be instructed to move quickly to the warm zone,
away from the immediate accident site to get decontaminated and processed
first. Those with minor injuries would be decontaminated next. Those with
more severe to most severe injuries would be treated in that order.

Nursing Process: Implementation

Client Need: Safe, Effective Care Environment

Cognitive Level: Application


Objective: Discuss situations requiring the need for client isolation or client
decontamination.

Strategy: Determine which client needs to be decontaminated first based on


reverse triage principles.

6. A preceptor is teaching a graduate nurse the concepts of mitigation. Which of


the following statements, if made by the graduate nurse, would indicate an
understanding of this concept?

Your Answer: “A key nursing activity related to mitigation is the active


participation in learning about the major aspects of
disasters.”

Rationale: Health care professionals are among the essential

Nursing Process: Planning

Client Need: Safe, Effective Care Environment

Cognitive Level: Application

Objective: Discuss the role of the nurse in disaster planning, response, and
mitigation.

Strategy: Understand and be able to define the concepts of disaster planning,


response, and mitigation. Utilize these definitions in order to select the
correct answer.

7. A military nurse is working in Iraq. Because of the potential threat of


hazardous gas, which of the following should be worn when working in a
dangerous war zone? Select all that apply.

Your Answer: A gas mask

Protective clothing

A hood, helmet, or headgear

Rationale: Gas masks are used in a broad range of military, industrial, and
emergency situations to protect the user from hazardous dust, gas, or other
aerosols. Biological contaminants that are spread through aerosolized
droplets create a threat to those not wearing personal protective equipment.
A gas mask may be considered as a high-performance respirator, usually
equipped with both eye protection and air supply protection or treatment. A
hood, helmet, or headgear is generally worn to protect the skin, eyes,
airways, and respiratory systems. Protective clothing is made to guard
against mild irritants to serious lethal materials. Some protective suits are
disposable, intended for one use only. Others are durable, multi-layered
fabrics, are completely impermeable and are reusable. Sunglasses and a
surgical mask will not provide enough protection in this instance.

Nursing Process: Planning

Client Need: Safe, Effective Care Environment

Cognitive Level: Application

Objective: Discuss the role of the nurse in disaster planning, response, and
mitigation.

Strategy: Determine if each item would be indicated in this situation. Multiple


answers are correct.

8. An emergency room nurse is at work when a major terrorist attack occurs. In


addition to caring for injured clients, the nurse must control the crowd. Which
of the following statements, if made by the nurse, demonstrates an
understanding of the concept of crowd control?

Your Answer: “The agency’s security personnel and/or the local police force
must control these crowds.”

Rationale: When a disaster occurs, many people converge on the site. Those
who come are the curious and those who truly mean to assist in the rescue
and recovery of victims. However, this crowd of people needs to be controlled
by authorities in charge of the site and rescue and recovery. #1 is incorrect
because the job of crowd control is not under the auspices of the nurse. # 2
is incorrect because chaos ensues when the crowd is not maintained. # 4 is
incorrect because nurses should not enter an area that has not been
secured.

Nursing Process: Planning

Client Need: Safe, Effective Care Environment

Cognitive Level: Application

Objective: Discuss the role of the nurse in disaster planning, response, and
mitigation.

Strategy: Determine if the principles of crowd control are demonstrated in


each answer choice. Eliminate each answer choice that incorrectly describes
crowd control.

9. A newly graduated nurse is learning about the nurse’s role in disaster relief as
part of an orientation to the hospital. Which of the following concepts is
accurate?
Your Answer: Nurses may have to assume expanded roles in making
decisions for the most appropriate treatment of casualties.

Rationale: Nurses must be aware of the roles nurses play in all aspects of
disaster preparedness and response. #1 is incorrect because learning about
disasters is essential. #2 is incorrect because the nurse’s role will be active,
not passive. #3 is incorrect because basic skills should be applied.

Nursing Process: Planning

Client Need: Safe, Effective Care Environment

Cognitive Level: Analysis

Objective: Discuss the role of the nurse in disaster planning, response, and
mitigation.

Strategy: Examine the role of the nurse in disaster planning.

10. A nursing student is learning about how to manage immunocompromised


clients in a disaster situation. Which of the following statements made by the
nursing student demonstrates an understanding of this concept?

Your Answer: “A compromised immune system may be due to treatments


such as chemotherapy, those who have had organ or bone
marrow transplants, or from an underlying disease such as
HIV.”

Rationale: Clients who are immunocompromised pose special problems to the


health care community especially if these persons are unable to access health
care quickly in a disaster situation. # 1 is incorrect because the risk is
greater. # 3 is incorrect because the vesicles or pustules will be distant from
the vaccination site. # 4 is incorrect because bone marrow transplant clients
need to avoid fresh fruits and vegetables.

Nursing Process: Assessment

Client Need: Safe, Effective Care Environment

Cognitive Level: Analysis

Objective: Identify ways that nurses are able to provide care to clients with
special considerations.

Strategy: Determine if each statement is correct with regard to caring for the
immunocompromised client.

1. The nurse is teaching a class on biological warfare. Which information should the nurseinclude in the
presentation?
a. Contaminated water is the only source of transmission of biological agents.

b. Vaccines are available and being prepared to counteract biological agents.

c. Biological weapons are less of a threat than chemical agents.

d. Biological weapons are easily obtained and result in significant mortality.

2. Which signs/symptoms would the nurse assess in the client who has been exposed to theanthrax
bacillus via the skin?

a. A scabby, clear fluid±filled vesicle.

b. Edema, pruritus, and a 2-mm ulcerated vesicle.

c. Irregular brownish-pink spots around the hairline.

d. Tiny purple spots flush with the surface of the skin.

3. The client has expired secondary to smallpox. Which information about funeral arrangementsis most
important for the nurse to provide to the client¶s family?

a. The client must be cremated.

b. Suggest an open casket funeral.

c. Bury the client within 24 hours.

d. Notify the public health department.

4. A chemical exposure has just occurred at an airport. An off-duty nurse, knowledgeable


aboutbiochemical agents, is giving directions to the travelers. Which direction should the nurseprovide
to the travelers?

a. Hold their breath as much as possible.

b. Stand up to avoid heavy exposure.

c. Lie down to stay under the exposure.

d. Attempt to breathe through their clothing.

5. The nurse is caring for a client in the prodromal phase of radiation exposure. Whichsigns/symptoms
would the nurse assess in the client?

a. Anemia, leukopenia, and thrombocytopenia.

b. Sudden fever, chills, and enlarged lymph nodes.


c. Nausea, vomiting, and diarrhea.

d. Flaccid paralysis, diplopia, and dysphagia.

6. The off-duty nurse hears on the television of a bioterrorism act in the community.Which action should
the nurse take first?

a. Immediately report to the hospital emergency room.

b. Call the American Red Cross to find out where to go.

c. Pack a bag and prepare to stay at the hospital.

d. Follow the nurse¶s hospital policy for responding.

7. Which situation would warrant the nurse obtaining information from a material safety datasheet
(MSDS)?

a. The custodian spilled a chemical solvent in the hallway.

b. A visitor slipped and fell on the floor that had just been mopped.

c. A bottle of antineoplastic agent broke on the client¶s floor.

d. The nurse was stuck with a contaminated needle in the client¶s room.

8. The triage nurse is working in the emergency department. Which client should be assessedfirst?

a. The 10-year-old child whose dad thinks the child¶s leg is broken.

b. The 45-year-old male who is diaphoretic and clutching his chest.

c. The 58-year-old female complaining of a headache and seeing spots.

d. The 25-year-old male who cut his hand with a hunting knife.

9. According to the North Atlantic Treaty Organization (NATO) triage system, which situationwould be
considered a level red (Priority 1)?

a. Injuries are extensive and chances of survival are unlikely.

b. Injuries are minor and treatment can be delayed hours to days.

c. Injuries are significant but can wait hours without threat to life or limb.

d. Injuries are life threatening but survivable with minimal interventions.

10. Which statement best describes the role of the medical-surgical nurse during a disaster?

a. The nurse may be assigned to ride in the ambulance.


b. The nurse may be assigned as a first assistant in the operating room.

c. The nurse may be assigned to crowd control.

d. The nurse may be assigned to the emergency department.Nursing

Board Exam Review Questions in Emergency Part 6/20(ANSWER KEY)

1. Answer: DRationale: Because of the variety of agents, the means of transmission, and lethality of
theagents, biological weapons, including anthrax, smallpox, and plague, is especially dangerous.

2. Answer: BRationale: Exposure to anthrax bacilli via the skin results in skin lesions, which cause
edemawith pruritus and the formation of macules or papules that ulcerate, forming a 1-3 mm
vesicle.Then a painless eschar develops, which falls off in one (1) to 2 weeks.

3. Answer: ARationale: Cremation is recommended because the virus can stay alive in the scabs of the
bodyfor 13 years.

4. Answer: BRationale: Standing up will avoid heavy exposure the chemical will sink toward the floor
orground.

5. Answer: CRationale: The prodromal phase (presenting symptoms) of radiation exposure occurs
48±72hours after exposure and the signs/symptoms are nausea, vomiting, diarrhea, anorexia,
andfatigue. Higher exposures of radiation signs/symptoms include fever, respiratory distress,
andexcitability.

6. Answer: DRationale: The nurse should follow the hospital¶s policy. Many times nurses will stay at
homeuntil decisions are made as to where the employees should report.

7. Answer: ARationale: The MSDS provides chemical information regarding specific agents,
healthinformation, and spill information for a variety of chemicals. It is required for every chemicalthat
is found in the hospital.

8. Answer: BRationale: The triage nurse should see this client first because these are symptoms of a
myocar-dial infarction, which potentially life is threatening.

9. Answer: DRationale: This is called the immediate category. Individuals in this group can progress
rapidlyto expectant if treatment is delayed.

10. Answer: DRationale: New settings and atypical roles for nurses may be required during disasters;
medical-surgical nurses can provide first aid and be required to work in unfamiliar settings.
Nursing Board Exam Review Questions in Emergency Part 5/20

1. Which intervention is the most important for the nurse to implement when performing mouth-to-
mouth resuscitation on a client who has pulseless ventricular fibrillation?

a. Perform the jaw thrust maneuver to open the airway.

b. Use the mouth to cover the client¶s mouth and nose.

c. Insert an oral airway prior to performing mouth to mouth.

d. Use a pocket mouth shield to cover client¶s mouth.

2. The nurse is teaching CPR to a class. Which statement best explains the definition of suddencardiac
death?

a. Cardiac death occurs after being removed from a mechanical ventilator.

b. Cardiac death is the time that the physician officially declares the client dead.

c. Cardiac death occurs within one (1) hour of the onset of cardiovascular symptoms.

d. The death is caused by myocardial ischemia resulting from coronary artery disease.

3. Which statement explains the scientific rationale for having emergency suction equipmentavailable
during resuscitation efforts?

a. Gastric distention can occur as a result of ventilation.

b. It is needed to assist when intubating the client.

c. This equipment will ensure a patent airway.

d. It keeps the vomitus away from the health-care provider.

4. Which equipment must be immediately brought to the client¶s bedside when a code is calledfor a
client who has experienced a cardiac arrest?

a. A ventilator.

b. A crash cart.

c. A gurney.

d. Portable oxygen
.5. The nursing administrator responds to a code situation. When assessing the situation, whichrole
must the administrator ensure is performed for legal purposes and continuity of care of theclient?

a. A person is ventilating with an ambu bag

.b. A person is performing chest compressions correctly.

c. A person is administering medications as ordered.

d. A person is keeping an accurate record of the code.

6. The nurse in the emergency department has admitted five (5) clients in the last two (2) hourswith
complaints of fever and gastrointestinal distress. Which question would be most appropriatefor the
nurse to ask each client to determine if there is a bioterrorism threat?

a. ³Do you work or live near any large power lines?´

b. ³Where were you immediately before you got sick?´

c. ³Can you write down everything you ate today?´

d. ³What other health problems do you have?

´7. The health-care facility has been notified that an alleged inhalation anthrax exposure hasoccurred at
the local post office. Which category of personal protective equipment (PPE) wouldthe response team
wear?

a. Level A

b. Level B

c. Level C

d. Level D

8. The nurse is teaching a class on bioterrorism and is discussing personal protective equipment(PPE).
Which statement is the most important fact that must be shared with the participants?

a. Health-care facilities should keep masks at entry doors

.b. The respondent should be trained in the proper use of PPE.

c. No single combination of PPE protects against all hazards.

d. The EPA has divided PPE into four levels of protection

9. The nurse is teaching a class on bioterrorism. What is the scientific rationale for designating aspecific
area for decontamination?
a. Showers and privacy can be provided to the client in this area.

b. This area isolates the clients who have been exposed to the agent

.c. It provides a centralized area for stocking the needed supplies.

d. It prevents secondary contamination to the health-care providers.

10. The triage nurse in a large trauma center has been notified of an explosion in a majorchemical
manufacturing plant. Which action should the nurse implement first when the clientsarrive at the
emergency department?

a. Triage the clients and send them to the appropriate areas.

b. Thoroughly wash the clients with soap and water and then rinse.

c. Remove the clients¶ clothing and have them shower.

d. Assume the clients have been decontaminated at the plant.

Nursing Board Exam Review Questions in Emergency Part 5/20 (ANSWER KEY)

1. Answer: DRationale: Nurses should protect themselves against possible communicable disease, such
asHIV, hepatitis, or any types of sexually transmitted disease.

2. Answer: CRationale: Unexpected death occurring within1 hour of the onset of cardiovascular
symptoms isthe definition of sudden cardiac death.

3. Answer: ARationale: Gastric distention occurs from overventilating clients. When compressions
areperformed, the pressure will cause vomiting that could be aspirated into the lungs.

4. Answer: BRationale: The crash cart is the mobile unit that has the defibrillator and all the medications
andsupplies needed to conduct a code.

5. Answer: DRationale: The chart is a legal document and the code must be documented in the chart
andprovide information that may be needed in the intensive care unit.

6. Answer: BRationale: The nurse should take note of any unusual illness for the time of year or clusters
ofclients coming from a single geographical location who all exhibit signs/symptoms of
possiblebiological terrorism.

7. Answer: ARationale: Level A protection is worn when the highest level of respiratory, skin, eye, and
mucous membrane protection is required.In this situation of possible inhalation of anthrax, such
protection is required.
8. Answer: CRationale: The health-care providers are not guaranteed absolute protects. The nurse
should takenote of any unusual illness for the time of year or clusters of clients coming from a
singlegeographical location who all exhibit signs/symptoms of possible biological terrorism.ion,
evenwith all the training and protective equipment.

9. Answer: DRationale: Avoiding cross contamination is a priority for personnel and equipment²the
fewernumber of people exposed, the safer the community and area.

10. Answer: CRationale: This is the first step. Depending on the type of exposure, this step alone can
remove alarge portion of exposure.

Nursing Board Exam Review Questions in Emergency Part 4/20

1. The nurse is planning a program for clients at a health fair regarding the prevention and
earlydetection of cancer of the pancreas. Which self-care activity should the nurse teach that is
anexample of primary nursing care?

a. Monitor for elevated blood glucose at random intervals.

b. Inspect the skin and sclera of the eyes for a yellow tint.

c. Limit meat in the diet and eat a diet that is low in fats.

d. Instruct the client with hyperglycemia about insulin injections.

2. The client diagnosed with cancer of the pancreas is being discharged to start chemotherapy inthe
HCP¶s office. Which statement made by the client indicates the client understands thedischarge
instructions?

a. ³I will have to see the HCP every day for six (6) weeks for my treatments.´

b. ³I should write down all my questions so I can ask them when I see the HCP.´

c. ³I am sure that this is not going to be a serious problem for me to deal with.´

d. ³The nurse will give me an injection in my leg and I will get to go home.´

3. The nurse caring for a client diagnosed with cancer of the pancreas writes the collaborativeproblem of
³altered nutrition.´ Which intervention should the nurse include in the plan of care?

a. Continuous feedings via PEG tube.

b. Have the family bring in foods from home.


c. Assess for food preferences.

d. Refer to the dietitian.

4. The client is taken to the emergency department with an injury to the left arm. Which actionshould
the nurse take first?

a. Assess the nail beds for capillary refill time.

b. Remove the client¶s clothing from the arm.

c. Call radiology for a STAT x-ray of the extremity.

d. Prepare the client for the application of a cast.

5. The nurse finds the client unresponsive on the floor of the bathroom. Which action should thenurse
implement first?

a. Check the client for breathing.

b. Assess the carotid artery for a pulse.

c. Shake the client and shout.

d. Call a code via the bathroom call light.

6. Which behavior by the unlicensed assistive personnel who is performing cardiac compressionson an
adult client during a code warrants immediate intervention by the nurse?

a. Has one hand on the lower half of the sternum above the xiphoid process.

b. Performs cardiac compressions and allows for rescue breathing.

c. Depresses the sternum 0.5 to one (1) inch during compressions.

d. Requests to be relieved from performing compressions because of exhaustion.

7. Which is the most important intervention for the nurse to implement when participating in acode?

a. Elevate the arm after administering medication.

b. Maintain sterile technique throughout the code.

c. Treat the client¶s signs/symptoms; do not watch the monitor.

d. Be sure to provide accurate documentation of what happened in the code.

8. The CPR instructor is explaining what an automated external defibrillator (AED) does tostudents in a
CPR class. Which statement best describes an AED?
a. It analyzes the rhythm and shocks the client in ventricular fibrillation.

b. The client will be able to have synchronized cardioversion with the AED.

c. It will keep the health-care provider informed of the client¶s oxygen level.

d. The AED will perform cardiac compressions on the client.

9. The nurse is caring for clients on a medical floor. Which client is most likely to experiencesudden
cardiac death?

a. The 84-year-old client exhibiting uncontrolled atrial fibrillation.

b. The 60-year-old client exhibiting asymptomatic sinus bradycardia.

c. The 53-year-old client exhibiting ventricular fibrillation.

d. The 65-year-old client exhibiting supraventricular tachycardia.

10. Which health-care team member referral should be made when a code is being conducted ona client
in a community hospital?

a. The hospital chaplain.

b. The social worker.

c. The respiratory therapist.

d. The director of nurses.

Nursing Board Exam Review Questions in Emergency Part 4/20 (ANSWER KEY)

1. Answer: CRationale: Limiting the intake of meat and fats in the diet would be an example of
primaryinterventions. Risk factors for the development of cancer of the pancreas are cigarette
smokingand eating a high-fat diet that is high in animal protein. By changing these behaviors the
clientcould possibly prevent the development of cancer of the pancreas. Other risk factors
includegenetic predisposition and exposure to industrial chemicals.

2. Answer: BRationale: The most important person in the treatment of the cancer is the client. Research
hasproved that the more involved a client becomes in his or her care, the better the prognosis.Clients
should have a chance to ask all the questions that they have.
3. Answer: DRationale: A collaborative intervention would be to refer to the nutrition expert, the
dietitian.

4. Answer: ARationale: The nurse should assess the nail beds for the capillary refill time. A prolonged
time(greater than three seconds) indicates impaired circulation to the extremity.

5. Answer: CRationale: This is the first intervention the nurse should implement after finding the
clientunresponsive on the floor.

6. Answer: CRationale: The sternum should be depressed 1.5 to 2 inches during compressions to
ensureadequate circulation of blood to the body; therefore, the nurse needs to correct the assistant

.7. Answer: CRationale: This is the most important intervention.The nurse should always treat the client
based on the nurse¶s assessment and data from themonitors; an intervention should not be based on
data from the monitors without the nurse¶sassessment.

8. Answer: ARationale: This is the correct statement explaining what an AED does when used in a code.

9. Answer: CRationale: Ventricular fibrillation is the most common dysrhythmia associated with
suddencardiac death; ventricular fibrillation is responsible for 65% to 85% of sudden cardiac deaths.

10. Answer: ARationale: The chaplain should be called to help address the client¶s family or significant
others.A small community hospital would not have a 24-hour on-duty pastoral service.

Nursing Board Sample Review Questions inEmergency22 Jul, 2010 | Written by Nursingbuzz_editor |
under Emergency Nursing Review Questions,Emergency Questions Nursing Board Exam Review
Questions in Emergency Part 3/20

1. A client with multiple injury following a vehicular accident is transferred to the critical careunit. He
begins to complain of increased abdominal pain in the left upper quadrant. A rupturedspleen is
diagnosed and he is scheduled for emergency splenectomy. In preparing the client forsurgery, the nurse
should emphasize in his teaching plan the:

a. Complete safety of the procedure

b. Expectation of postoperative bleeding

c. Risk of the procedure with his other injuries

d. Presence of abdominal drains for several days after surgery

2. After you managed to stabilize the respiratory function of your burn patient, your next goal isto
prevent this you have to replace the lost fluid and electrolytes. In starting fluid replacementtherapy, the
total volume and rate of IV fluid repalcement are gauged by the patient¶s responseand by the patient¶s
response and by the resuscitation formula. In determining the adequacy offluid resuscitation, it is
essential for you to monitor the:

a. urine output

b. blood pressure

c. intracranial pressure

d. cardiac output

3. You are a nurse in the emergency department and it is during the shift that Mr. CT is admittedin the
area due to a fractured skull from a motor accident. You scheduled him for surgery underwhich
classification?

a. Urgent

b. Emergent

c. Required

d. Elective

4. Lucky was in a vehicular acccident where he sustained injury to his left ankle. In theEmergency room,
you noticed anxious he looks. You establish rapport with him and to reducehis anxiety, you initially:

a. Identify yourself and state your purpose in being with the client

b. Take him to the radiology section for x-ray of affected extremity

c. Talk to the physician for an order of valium

d. Do inspection and palpation to check extent of his injuries

5. The client diagnosed with a mild concussion is being discharged from the emergencydepartment.
Which discharge instruction should the nurse teach the client¶s significant other?

a. Awaken the client every two hours.

b. Monitor for increased intracranial pressure.

c. Observe frequently for hypervigilance.

d. Offer the client food every three to four hours.

6. The client diagnosed with Addison¶s disease is admitted to the emergency department after aday at
the lake. The client is lethargic, forgetful, and weak. Which intervention should be theemergency
department nurse¶s first action?
a. Start an IV with an 18-gauge needle and infuse NS rapidly

.b. Have the client wait in the waiting room until a bed is available

.c. Perform a complete head-to-toe assessment.

d. Collect urinalysis and blood samples for a CBC and calcium level.

7. The nurse caring for a client diagnosed with cancer of the pancreas writes the nursingdiagnosis of
³risk for altered skin integrity related to pruritus.´ Which interventions should thenurse implement?

a. Assess tissue turgor.

b. Apply antifungal creams.

c. Monitor bony prominences for breakdown.

d. Have the client keep the fingernails short.

8. The client diagnosed with cancer of the head of the pancreas is two (2)
dayspostpancreatoduodenectomy (Whipple¶s procedure). Which nursing problem has the
highestpriority?

a. Anticipatory grieving.

b. Fluid volume imbalance.

c. Acute incisional pain.

d. Altered nutrition.

9. The client is diagnosed with cancer of the head of the pancreas. When assessing the patient,which
signs and symptoms would the nurse expect to find?

a. Clay-colored stools and dark urine

.b. Night sweats and fever.

c. Left lower abdominal cramps and tenesmus.

d. Nausea and coffee-ground emesis.

10. The client admitted to rule out pancreatic islet tumors complains of feeling weak, shaky, andsweaty.
Which should be the first intervention implemented by the nurse?

a. Start an IV with D5W.

b. Notify the health-care provider.


c. Perform a bedside glucose check.

d. Give the client some orange juice.

Nursing Board Exam Review Questions in Emergency Part 3/20 (ANSWER KEY)

1. Answer: DRationale: Presence of abdominal drains for several days after surgeryDrains are usually
inserted into the splenic bed to facilitate removal of fluid in the area that couldlead to abscess
formation.

2. Answer: ARationale: to establish the sufficiency of fluid resuscitation, urine output totals an index of
renalperfusion. Urine output totals an index of renal perfusion, urine output totals of 30-50
ml/hourhave been used as resuscitation goals. Other indicators of adequate fluid replacement are
systolicblood pressure exceeding 100 mmHg, a pulse rate less than110 beats/min or both.

3. Answer: BRationale: Emergent surgery is performed, immediately without delay to maintain life, limb
ororgan, remove damage and stop bleeding. Urgent surgery requires prompt attention and is donefew
hours but within 24 to 48 hours. Required surgery is done within a few weeks as surgery isimportant.
Elective surgery is scheduled and done at the convenience of client as failure to havesurgery is not
catastrophic. Optional surgeries are done by preference only.

4. Answer: ARationale: Introducing self initiates the nurse-patient interaction, relationship and the
purpose ofbeing with the client. This prevents confusion and let the client know what to expect,
therebyreducing anxiety

.5. Answer: ARationale: Awakening the client every 2 hours allows the identification of headache,
dizziness,lethargy, irritability, and anxiety²all signs of post-concussion syndrome²that would warrantthe
significant other¶s taking the client back to the emergency department.

6. Answer: ARationale: This client has been exposed to wind and sun at the lake during the hours prior
tobeing admitted to the emergency department. This predisposes the client to dehydration and
anAddisonian crisis. Rapid IV fluid replacement is necessary

.7. Answer: DRationale: Keeping the fingernails short will reduce the chance of breaks in the skin
fromscratching.

8. Answer: BRationale: This is a major abdominal surgery, and there are massive fluid volume shifts
thatoccur when this type of trauma is experienced by the body. Maintaining the circulatory
systemwithout overloading it requires extremely close monitoring.

9. Answer: ARationale: The client will have jaundice, clay-colored stools, and tea-colored urine resulting
fromblockage of the bile drainage
.10. Answer: CRationale: These are symptoms of an insulin reaction (hypoglycemia). A bedside glucose
checkshould be done. Pancreatic islet tumors can produce hyperinsulinemia or hypoglycemia.

Nursing Board Exam Review Questions in Emergency Part 2/20

1 Which nursing intervention would be appropriate when caring for a client who has sustained
anelectrical burn?

a. Applying ice to the burned area

b. Flushing the burn area with large amounts of water

c. Monitoring the client with cardiac telemetryd

. Preparing to administer the chemical antidote

2. Eddie, 40 years old, is brought to the emergency room after the crash of his private plane. Hehas
suffered multiple crushing wounds of the chest, abdomen and legs. It is feared his leg mayhave to be
amputated.When Eddie arrives in the emergency room, the assessment that assume the greatest
priority are:

a. Level of consciousness and pupil size

b. Abdominal contusions and other wounds

c. Pain, Respiratory rate and blood pressure

d. Quality of respirations and presence of pulses.

3. An emergency treatment for an acute asthmatic attack is Adrenaline 1:1000 givenhypodermically.


This is given to:

a. increase BP

b. decrease mucosal swelling

c. relax the bronchial smooth muscle

d. decrease bronchial secretions

4. Intervention for a pt. who has swallowed a Muriatic Acid includes all of the following except

a. administering an irritant that will stimulate vomiting

b. aspirating secretions from the pharynx if respirations are affected

c. neutralizing the chemical


d. washing the esophagus with large volumes of water via gastric lavage

5. John, 16 years old, is brought to the ER after a vehicular accident. He is pronounced dead onarrival.
When his parents arrive at the hospital, the nurse should:

a. ask them to stay in the waiting area until she can spend time alone with them

b. speak to both parents together and encourage them to support each other and express theiremotions
freely

c. Speak to one parent at a time so that each can ventilate feelings of loss without upsetting the other

d. ask the MD to medicate the parents so they can stay calm to deal with their son¶s death

.6. A nurse is eating in the hospital cafeteria when a toddler at a nearby table chokes on a piece offood
and appears slightly blue. The appropriate initial action should be to

a. Begin mouth to mouth resuscitation

b. Give the child water to help in swallowing

c. Perform 5 abdominal thrusts

d. Call for the emergency response team

7. A client is admitted from the emergency department with severe-pain and edema in the rightfoot. His
diagnosis is gouty arthritis. When developing a plan of care, which action would havethe highest
priority?

a. Apply hot compresses to the affected joints.

b. Stress the importance of maintaining good posture to prevent deformities

.c. Administer salicylates to minimize the inflammatory reaction.

d. Ensure an intake of at least 3000 ml of fluid per day.

8. The Heimlich maneuver (abdominal thrust), for acute airway obstruction, attempts to

:a. Force air out of the lungs

b. Increase systemic circulation

c. Induce emptying of the stomach

d. Put pressure on the apex of the heart

9. A nurse is performing CPR on an adult patient. When performing chest compressions, thenurse
understands the correct hand placement is located over the
a. upper half of the sternum

b. upper third of the sternum

c. lower half of the sternum

d. lower third of the sternum

John, 16 years old, is brought to the ER after a vehicular accident. He is pronounced dead onarrival.
When his parents arrive at the hospital, the nurse should:

a. ask them to stay in the waiting area until she can spend time alone with them

b. speak to both parents together and encourage them to support each other and express theiremotions
freely

c. Speak to one parent at a time so that each can ventilate feelings of loss without upsetting theother

d. ask the MD to medicate the parents so they can stay calm to deal with their son¶s death.

Nursing Board Exam Review Questions in Emergency Part 2/20 (ANSWER KEY)

1. Answer: CRationale: Because of the effects of the electrical current on the cardiovascular system, all
clientsexperiencing electrical burns should be placed on a cardiac monitor. Applying ice is inappropriate
for any type of burn. Only chemical burns should be flushed with large amounts ofwater. Chemical
antidotes may be used for chemical burns for which an antidote has beenidentified.

2. Answer: DRationale: Respiratory and cardiovascular functions are essential for oxygenation. These are
toppriorities to trauma management. Basic life functions must be maintained or reestablished

3. Answer: CRationale: Acute asthmatic attack is characterized by severe bronchospasm which can
berelieved by the immediate administration of bronchodilators. Adrenaline or Epinephrine is
anadrenergic agent that causes bronchial dilation by relaxing the bronchial smooth muscles.

4. Answer: ARationale: Swallowing of corrosive substances causes severe irritation and tissue
destruction ofthe mucous membrane of the GI tract. Measures are taken to immediately remove the
toxin orreduce its absorption. For corrosive poison ingestion, such as in muriatic acid where burn
orperforation of the mucosa may occur, gastric emptying procedure is immediately instituted,
Thisincludes gastric lavage and the administration of activated charcoal to absorb the
poison.Administering an irritant with the concomitant vomiting to remove the swallowed poison
willfurther cause irritation and damage to the mucosal lining of the digestive tract. Vomiting is
onlyindicated when non-corrosive poison is swallowed.

5. Answer: BRationale: Sudden death of a family member creates a state of shock on the family. They go
intoa stage of denial and anger in their grieving. Assisting them with information they need to
know,answering their questions and listening to them will provide the needed support for them to
moveon and be of support to one another.

6. Answer: CRationale: Perform 5 abdominal thrusts. At this age, the most effective way to clear the
airway offood is to perform abdominal thrusts.

7. Answer: DRationale: Ensure an intake of at least 3000 ml of fluid per day. Gouty arthritis is a
metabolicdisease marked by urate deposits that cause painful arthritic joints. The patient should be
urgedto increase his fluid intake to prevent the development of urinary uric acid stones.

8. Answer: ARationale: The Heimlich maneuver is used to assist a person choking on a foreign object.
Thepressure from the thrusts lifts the diaphragm, forces air out of the lungs and creates an
artificialcough that expels the aspirated material.

9. Answer: CRationale: The exact and safe location to do cardiac compression is the lower half of the15.
sternum. Doing it at the lower third of the sternum may cause gastric compression which canlead to a
possible aspiration.

10. Answer: BRationale: Sudden death of a family member creates a state of shock on the family. They
go intoa stage of denial and anger in their grieving. Assisting them with information they need to
know,answering their questions and listening to them will provide the needed support for them to
moveon and be of support to one another.
1. You are the charge nurse in an emergency department (ED) and must assign two staff
members to cover the triage area. Which team is the most appropriate for this
assignment?

a. An advanced practice nurse and an experienced LPN/LVN


b. An experienced LPN/LVN and an inexperienced RN
c. An experienced RN and an inexperienced RN
d. An experienced RN and a nursing assistant

2. You are working in the triage area of an ED, and four patients approach the triage desk
at the same time. List the order in which you will assess these patients.

a. An ambulatory, dazed 25-year-old male with a bandaged head wound


b. An irritable infant with a fever, petechiae, and nuchal rigidity
c. A 35-year-old jogger with a twisted ankle, having pedal pulse and no deformity
d. A 50-year-old female with moderate abdominal pain and occasional vomiting
_____, _____, _____, _____
3. In conducting a primary survey on a trauma patient, which of the following is considered
one of the priority elements of the primary survey?

a. Complete set of vital signs


b. Palpation and auscultation of the abdomen
c. Brief neurologic assessment
d. Initiation of pulse oximetry

4. A 56-year-old patient presents in triage with left-sided chest pain, diaphoresis, and
dizziness. This patient should be prioritized into which category?

a. High urgent
b. Urgent
c. Non-urgent
d. Emergent

5. The physician has ordered cooling measures for a child with fever who is likely to be
discharged when the temperature comes down. Which of the following would be appropriate
to delegate to the nursing assistant?

a. Assist the child to remove outer clothing.


b. Advise the parent to use acetaminophen instead of aspirin.
c. Explain the need for cool fluids.
d. Prepare and administer a tepid bath.

6. It is the summer season, and patients with signs and symptoms of heat-related illness
present in the ED. Which patient needs attention first?

a. An elderly person complains of dizziness and syncope after standing in the sun for several
hours to view a parade
b. A marathon runner complains of severe leg cramps and nausea. Tachycardia,
diaphoresis, pallor, and weakness are observed.
c. A previously healthy homemaker reports broken air conditioner for days. Tachypnea,
hypotension, fatigue, and profuse diaphoresis are observed.
d. A homeless person, poor historian, presents with altered mental status, poor muscle
coordination, and hot, dry, ashen skin. Duration of exposure is unknown.

7. You respond to a call for help from the ED waiting room. There is an elderly patient lying
on the floor. List the order for the actions that you must perform.

a. Perform the chin lift or jaw thrust maneuver.


b. Establish unresponsiveness.
c. Initiate cardiopulmonary resuscitation (CPR).
d. Call for help and activate the code team.
e. Instruct a nursing assistant to get the crash cart.
_____, _____, _____, _____, _____

8. The emergency medical service (EMS) has transported a patient with severe chest pain.
As the patient is being transferred to the emergency stretcher, you note unresponsiveness,
cessation of breathing, and no palpable pulse. Which task is appropriate to delegate to the
nursing assistant?

a. Chest compressions
b. Bag-valve mask ventilation
c. Assisting with oral intubation
d. Placing the defibrillator pads

9. An anxious 24-year-old college student complains of tingling sensations, palpitations, and


chest tightness. Deep, rapid breathing and carpal spasms are noted. What priority nursing
action should you take?

a. Notify the physician immediately.


b. Administer supplemental oxygen.
c. Have the student breathe into a paper bag.
d. Obtain an order for an anxiolytic medication.

10.An experienced traveling nurse has been assigned to work in the ED; however, this is
the nurse’s first week on the job. Which area of the ED is the most
appropriate assignment for the nurse?

a. Trauma team
b. Triage
c. Ambulatory or fats track clinic
d. Pediatric medicine team

11.A tearful parent brings a child to the ED for taking an unknown amount of children’s
chewable vitamins at an unknown time. The child is currently alert and
asymptomatic. What information should be immediately reported to the physician?

a. The ingested children’s chewable vitamins contain iron.


b. The child has been treated several times for ingestion of toxic substances.
c. The child has been treated several times for accidental injuries.
d. The child was nauseated and vomited once at home.

12.In caring for a victim of sexual assault, which task is most appropriate for an LPN/LVN?

a. Assess immediate emotional state and physical injuries


b. Collect hair samples, saliva swabs, and scrapings beneath fingernails.
c. Provide emotional support and supportive communication.
d. Ensure that the “chain of custody” is maintained.

13.You are caring for a victim of frostbite to the feet. Place the following interventions in the
correct order.
a. Apply a loose, sterile, bulky dressing.
b. Give pain medication.
c. Remove the victim from the cold environment.
d. Immerse the feet in warm water 100o F to 105o F (40.6o C to 46.1o C)
_____, _____, _____, _____

14.A patient sustains an amputation of the first and second digits in a chainsaw accident.
Which task should be delegated to the LPN/LVN?

a. Gently cleanse the amputated digits with Betadine solution.


b. Place the amputated digits directly into ice slurry.
c. Wrap the amputated digits in sterile gauze moistened with saline.
d. Store the amputated digits in a solution of sterile normal saline.

15.A 36-year-old patient with a history of seizures and medication compliance of phenytoin
(Dilantin) and carbamazepine (Tegretol) is brought to the ED by the MS personnel for
repetitive seizure activity that started 45 minutes prior to arrival. You anticipate that the
physician will order which drug for status epilepticus?

a. PO phenytoin and carbamazepine


b. IV lorazepam (Ativan)
c. IV carbamazepam
d. IV magnesium sulfate

16.You are preparing a child for IV conscious sedation prior to repair of a facial laceration.
What information should you immediately report to the physician?

a. The parent is unsure about the child’s tetanus immunization status.


b. The child is upset and pulls out the IV.
c. The parent declines the IV conscious sedation.
d. The parent wants information about the IV conscious sedation.

17.An intoxicated patient presents with slurred speech, mild confusion, and uncooperative
behavior. The patient is a poor historian but admits to “drinking a
few on the weekend.” What is the priority nursing action for this patient?

a. Obtain an order for a blood alcohol level.


b. Contact the family to obtain additional history and baseline information.
c. Administer naloxone (Narcan) 2 - 4 mg as ordered.
d. Administer IV fluid support with supplemental thiamine as ordered.

18.When an unexpected death occurs in the ED, which of the following tasks is most
appropriate to delegate to the nursing assistant?

a. Escort the family to a place of privacy.


b. Go with the organ donor specialist to talk to the family.
c. Assist with postmortem care.
d. Assist the family to collect belongings.

19.Following emergency endotracheal intubation, you must verify tube placement and
secure the tube. List in order the steps that are required to perform this
function?

a. Obtain an order for a chest x-ray to document tube placement.


b. Secure the tube in place.
c. Auscultate the chest during assisted ventilation.
d. Confirm that the breath sounds are equal and bilateral.
_____, _____, _____, _____

20.A teenager arrives by private car. He is alert and ambulatory, but this shirt and pants
are covered with blood. He and his hysterical friends are yelling and trying to explain that
that they were goofing around and he got poked in the abdomen with a stick. Which of the
following comments should be given first
consideration?

a. “There was a lot of blood and we used three bandages.”


b. “He pulled the stick out, just now, because it was hurting him.”
c. “The stick was really dirty and covered with mud.”
d. “He’s a diabetic, so he needs attention right away.”

21.A prisoner, with a known history of alcohol abuse, has been in police custody for 48
hours. Initially, anxiety, sweating, and tremors were noted. Now,
disorientation, hallucination, and hyper-reactivity are observed. The medical diagnosis is
delirium tremens. What is the priority nursing diagnosis?

a. Risk for Injury related to seizures


b. Risk for Other-Directed Violence related to hallucinations
c. Risk for Situational Low Self-esteem related to police custody
d. Risk for Nutritional Deficit related to chronic alcohol abuse

22.You are assigned to telephone triage. A patient who was stung by a common honey bee
calls for advice, reports pain and localized swelling, but denies any
respiratory distress or other systemic signs of anaphylaxis. What is the action that you
should direct the caller to perform?

a. Call 911.
b. Remove the stinger by scraping.
c. Apply a cool compress.
d. Take an oral antihistamine.

23.In relation to submersion injuries, which task is most appropriate to delegate to an


LPN/LVN?
a. Talk to a community group about water safety issues.
b. Stabilize the cervical spine for an unconscious drowning victim.
c. Remove wet clothing and cover the victim with a warm blanket.
d. Monitor an asymptomatic near-drowning victim.

24.You are assessing a patient who has sustained a cat bite to the left hand. The cat is up-
to-date immunizations. The date of the patient’s last tetanus shot is unknown. Which of the
following is the priority nursing diagnosis?

a. Risk for Infection related to organisms specific to cat bites


b. Impaired Skin Integrity related to puncture wounds
c. Ineffective Health Maintenance related to immunization status
d. Risk for Impaired Mobility related to potential tendon damage

25.These patients present to the ED complaining of acute abdominal pain. Prioritize them in
order of severity.

a. A 35-year-old male complaining of severe, intermittent cramps with three episodes of


watery diarrhea, 2 hours after eating
b. A 11-year-old boy with a low-grade fever, left lower quadrant tenderness, nausea, and
anorexia for the past 2 days
c. A 40-year-old female with moderate left upper quadrant pain, vomiting small amounts of
yellow bile, and worsening symptoms over the past week
d. A 56-year-old male with a pulsating abdominal mass and sudden onset of pressure-like
pain in the abdomen and flank within the past hour
_____, _____, _____, _____

26.The nursing manager decides to form a committee to address the issue of violence
against ED personnel. Which combination of employees is best suited to
fulfill this assignment?

a. ED physicians and charge nurses


b. Experienced RNs and experienced paramedics
c. RNs, LPN/LVNs, and nursing assistants
d. At least one representative from each group of ED personnel

27.In a multiple-trauma victim, which assessment finding signals the most serious and life-
threatening condition?

a. A deviated trachea
b. Gross deformity in a lower extremity
c. Decreased bowel sounds
d. Hematuria

28.A patient in a one-car rollover presents with multiple injuries. Prioritize the interventions
that must be initiated for this patient.
a. Secure/start two large-bore IVs with normal saline
b. Use the chin lift or jaw thrust method to open the airway.
c. Assess for spontaneous respirations
d. Give supplemental oxygen per mask.
e. Obtain a full set of vital signs.
f. Remove patient’s clothing.
g. Insert a Foley catheter if not contraindicated.
_____, _____, _____, _____, ____, ____, ____

29.In the work setting, what is your primary responsibility in preparing for disaster
management that includes natural disasters or bioterrorism incidents?

a. Knowledge of the agency’s emergency response plan


b. Awareness of the signs and symptoms for potential agnets of bioterrorism
c. Knowledge of how and what to report to the CDC
d. Ethical decision-making about exposing self to potentially lethal substances

30.You are giving discharge instructions to a woman who has been treated for contusions
and bruises sustained during an episode of domestic violence. What
is your priority intervention for this patient?

a. Transportation arrangements to a safe house


b. Referral to a counselor
c. Advise about contacting the police
d. Follow-up appointment for injuries

RATIONALE
MEDICAL – SURGICAL EMERGENCIES

1. ANSWER C – Triage requires at least one experienced RN. Pairing an experienced RN with
inexperienced RN provides opportunities for mentoring. Advanced practice nurses are
qualified to perform triage; however, their services are usually required in other areas of the
ED. An LPN/LVN is not qualified to perform the initial patient assessment or decision
making. Pairing an experienced RN with a nursing assistant is the second best option,
because the assistant can obtain vital signs and assist in transporting.

2. ANSWER B, A, D, C – An irritable infant with fever and petechiae should be further


assessed for other meningeal signs. The patient with the head wound needs additional
history and assessment for intracranial pressure. The patient with moderate abdominal pain
is uncomfortable, but not unstable at this point. For the ankle injury, medical evaluation can
be delayed 24 – 48 hours if necessary.
3. ANSWER C – A brief neurologic assessment to determine level of consciousness and pupil
reaction is part of the primary survey. Vital signs, assessment of the abdomen, and
initiation of pulse oximetry are considered part of the secondary survey.

4. ANSWER D – Chest pain is considered an emergent priority, which is defined as


potentially life-threatening. Patients with urgent priority need treatment within 2 hours of
triage (e.g. kidney stones). Non-urgent conditions can wait for hours or even days. (High
urgent is not commonly used; however, in 5-tier triage systems, High urgent patients fall
between emergent and urgent in terms of the time lapsing prior to treatment).

5. ANSWER A – The nursing assistant can assist with the removal of the outer clothing,
which allows the heat to dissipate from the child’s skin. Advising and
explaining are teaching functions that are the responsibility of the RN. Tepid baths are not
usually performed because of potential for rebound and shivering.

6. ANSWER D – The homeless person has symptoms of heat stroke, a medical emergency,
which increases risk for brain damage. Elderly patients are at risk for
heat syncope and should be educated to rest in cool area and avoid future similar situations.
The runner is having heat crams, which can be managed with rest and fluids. The housewife
is experiencing heat exhaustion, and management includes fluids (IV or parenteral) and
cooling measures. The prognosis for recovery is good.

7. ANSWER B, D, A, C, E – Establish unresponsiveness first. (The patient may have fallen


and sustained a minor injury.) If the patient is unresponsive, get help and have someone
initiate the code. Performing the chin lift or jaw thrust maneuver opens the airway. The
nurse is then responsible for starting CPR. CPR should not be interrupted until the patient
recovers or it is determined that heroic efforts have been exhausted. A crash cart should be
at the site when the code team arrives; however, basic CPR can be effectively performed
until the team arrives.

8. ANSWER A – Nursing assistants are trained in basic cardiac life support and can perform
chest compressions. The use of the bag-valve mask requires practice and usually a
respiratory therapist will perform this function. The nurse or the respiratory therapist should
provide PRN assistance during intubation. The defibrillator pads are clearly marked;
however, placement should be done by the RN or physician because of the potential for skin
damage and electrical arcing.

9. ANSWER C – The patient is hyperventilating secondary to anxiety, and breathing into a


paper bag will allow rebreathing of carbon dioxide. Also, encouraging slow breathing will
help. Other treatments such as oxygen and medication may be needed if other causes are
identified.

10. ANSWER C – The fast track clinic will deal with relatively stable patients. Triage, trauma,
and pediatric medicine should be staffed with experienced nurses who know the hospital
routines and policies and can rapidly locate equipment.
11. ANSWER A – Iron is a toxic substance that can lead to massive hemorrhage, coma,
shock, and hepatic failure. Deferoxame is an antidote that can be used for severe cases of
iron poisoning. Other information needs additional investigation, but will not change the
immediate diagnostic testing or treatment plan.

12. ANSWER C – The LPN/LVN is able to listen and provide emotional support for her
patients. The other tasks are the responsibility of an RN or, if available, a SANE (sexual
assault nurse examiner) who has received training to assess, collect and safeguard
evidence, and care for these victims.

13. ANSWER C, B, D, A – The victim should be removed from the cold environment first,
and then the rewarming process can be initiated. It will be painful, so give pain medication
prior to immersing the feet in warmed water.

14. ANSWER C – The only correct intervention is C. the digits should be gently cleansed
with normal saline, wrapped in sterile gauze moistened with saline, and placed in a plastic
bag or container. The container is then placed on ice.

15. ANSWER B – IV Lorazepam (Ativan) is the drug of choice for status epilepticus. Tegretol
is used in the management of generalized tonic-clonic, absence or mixed type seizures, but
it does not come in an IV form. PO (per os) medications are inappropriate for this
emergency situation. Magnesium sulfate is given to control seizures in toxemia of
pregnancy.

16. ANSWER C – Parent refusal is an absolute contraindication; therefore, the physician


must be notified. Tetanus status can be addressed later. The RN can
restart the IV and provide information about conscious sedation; if the parent still
notsatisfied, the physician can give more information.

17. ANSWER D – The patient presents with symptoms of alcohol abuse and there is a risk
for Wernicke’s syndrome, which is caused by a thiamine deficiency. Multiples drug abuse is
not uncommon; however, there is nothing in the question that suggests an opiate overdose
that requires naloxone. Additional information or the results of the blood alcohol level are
part of the total treatment plan but should not delay the immediate treatment.

18. ANSWER C – Postmortem care requires some turning, cleaning, lifting, etc., and the
nursing assistant is able to assist with these duties. The RN should take responsibility for
the other tasks to help the family begin the grieving process. In cases of questionable
death, belongings may be retained for evidence, so the chain of custody would have to be
maintained.

19. ANSWER C, D, B, A – Auscultating and confirming equal bilateral breath sounds should
be performed in rapid succession. If the sounds are not equal or if the sounds are heard
over the mid-epigastric area, tube placement must be corrected immediately. Securing the
tube is appropriate while waiting for the x-ray study.
20. ANSWER B – An impaled object may be providing a tamponade effect, and removal can
precipitate sudden hemodynamic decompensation. Additional history including a more
definitive description of the blood loss, depth of penetration, and medical history should be
obtained. Other information, such as the dirt on the stick or history of diabetes, is important
in the overall treatment plan, but can be addressed later.

21. ANSWER A – The patient demonstrates neurologic hyperactivity and is on the verge of a
seizure. Patient safety is the priority. The patient needs chlordiazepoxide (Librium) to
decrease neurologic irritability and phenytoin (Dilantin) for seizures. Thiamine and
haloperidol (Haldol) will also be ordered to address the other problems. The other diagnoses
are pertinent but not as immediate.

22. ANSWER B – The stinger will continue to release venom into the skin, so prompt
removal of the stinger is advised. Cool compresses and antihistamines can follow. The caller
should be further advised about symptoms that require 911 assistance.

23. ANSWER D – The asymptomatic patient is currently stable but should be observed for
delayed pulmonary edema, cerebral edema, or pneumonia. Teaching and care of critical
patients is an RN responsibility. Removing clothing can be delegated to a nursing assistant.

24. ANSWER A – Cat’s mouths contain a virulent organism, Pasteurella multocida, that can
lead to septic arthritis or bacteremia. There is also a risk for tendon damage due to deep
puncture wounds. These wounds are usually not sutured. A tetanus shot can be given
before discharge.

25. ANSWER D, B, C, A – The patient with a pulsating mass has an abdominal aneurysm
that may rupture and he may decompensate suddenly. The 11-year-old boy needs
evaluation to rule out appendicitis. The woman needs evaluation for gallbladder problems
that appear to be worsening. The 35-year-old man has food poisoning, which is usually self-
limiting.

26. ANSWER D – At least one representative from each group should be included because
all employees are potential targets fro violence in the ED.

27. ANSWER A – A deviated trachea is a symptoms of tension pneumothorax. All of the


other symptoms need to be addressed, but are of lesser priority.

28. ANSWER C, B, D, A, E, F, G – For a multiple trauma victim, many interventions will


occur simultaneously as team members assist in the resuscitation. Methods to open the
airway such as the chin lift or jaw thrust can be used simultaneously while assessing for
spontaneous respirations. However, airway and oxygenation are priority. Starting IVs for
fluid resuscitation is part of supporting circulation. (EMS will usually establish at least one IV
in the field.) Nursing assistants can be directed to take vitals and remove clothing. Foley
catheter is necessary to closely monitor output.

29. ANSWER A – In preparing for disasters, the RN should be aware of the emergency
response plan. The plan gives guidance that includes roles of team members,
responsibilities, and mechanisms of reporting. Signs and symptoms of many agents will
mimic common complaints, such as flu-like symptoms. Discussions with colleagues and
supervisors may help the individual nurse to sort through ethical dilemmas related to
potential danger to self.

30. ANSWER A – Safety is a priority for this patient, and she should not return to a place
where violence could reoccur. The other options are important for the long term
management of this care.

Terms Definitions

Observe the patient's respiratory effort. During the primary assessment of a


trauma victim, the nurse determines
rationale: that the patient is breathing and has
Even with a patent airway, patients can an unobstructed airway. Which action
have other problems that compromise should the nurse take next?
ventilation, so the next action is to assess
the patient's breathing. The other actions
also are part of the initial survey but
assessment of breathing should be done
immediately after assessing for airway
patency.

Initiate isotonic fluid infusion through two During the primary survey of a
large-bore IV lines. patient with multiple traumatic
injuries, the nurse observes that the
rationale: patient's right pedal pulses are
The assessment data indicate that the absent and the leg is swollen. Which
patient may have arterial trauma and of these actions will the nurse take
hemorrhage. When a possibly life- next?
threatening injury is found during the
primary survey, the nurse should
immediately start interventions before
proceeding with the survey. Although a
CBC is indicated, administration of IV fluids
should be started first. Completion of the
primary survey and further assessment
should be completed after the IV fluids are
initiated.

Rapidly infuse cold normal saline. After resuscitation, a patient who


had a cardiac arrest is nonresponsive
rationale: to commands and therapeutic
When therapeutic hypothermia is used hypothermia is prescribed. Which
postresuscitation, cold normal saline is action will the nurse include in the
infused to rapidly lower body temperature plan of care?
to 89.6° F to 93.2° F (32° C to 34° C). Since
hypothermia will decrease brain activity,
neurologic assessment every 30 minutes is
not needed. Sedative medications are
administered during therapeutic
hypothermia.

obtain a Glasgow Coma Scale score. A patient who is unconscious after a


fall from a ladder is transported to
rationale: the emergency department by family
The Glasgow Coma Scale is included when members. During the primary survey
assessing for disability during the primary of the patient, the nurse should
survey. The other information is part of the _____________
secondary survey.

TIG and tetanus-diphtheria toxoid and An 18-year-old is brought to the


pertussis vaccine (Tdap). emergency department (ED) with
multiple lacerations and tissue
rationale: avulsion of the right hand. When
For an adult with no previous tetanus asked about tetanus immunization,
immunizations, TIG and Tdap are the patient denies having any
recommended. The other immunizations previous vaccinations. The nurse will
are not sufficient for this patient. anticipate administration of
______________

ultrasonography. A patient who has experienced blunt


abdominal trauma during a car
rationale: accident is complaining of increasing
For patients who are at risk for abdominal pain. The nurse will plan
intraabdominal bleeding, focused to teach the patient about the
abdominal ultrasonography is the preferred purpose of ______________
method to assess for intraperitoneal
bleeding. An MRI would not be used.
Peritoneal lavage is an alternative, but it is
more invasive. An NG tube would not be
helpful in diagnosis of intraabdominal
bleeding.
"I should have sports drinks when A patient with hypotension and
exercising outside in hot weather." temperature elevation after doing
yard work on a hot day is treated in
rationale: the ED. After the nurse has
Electrolyte solutions such as sports drinks completed discharge teaching, which
help replace fluid and electrolytes lost statement by the patient indicates
when exercising in hot weather. Salt tablets that the teaching has been effective?
are not recommended because of the risks
of gastric irritation and hypernatremia.
Antipyretic medications are not effective in
lowering body temperature elevations
caused by excessive exposure to heat. A
patient who is confused is likely to have
more severe hyperthermia and will be
unable to remember to take appropriate
action.

attach a cardiac monitor. When preparing to rewarm a patient


with hypothermia, the nurse will plan
rationale: to _____________
Rewarming can produce dysrhythmias, so
the patient should be monitored and
treated if necessary. Urinary catheterization
and endotracheal intubation are not
needed for rewarming. Sympathomimetic
drugs tend to stimulate the heart and
increase the risk for fatal dysrhythmias
such as ventricular fibrillation.

Auscultate breath sounds. A patient who experienced a near


drowning accident in a local lake, but
rationale: now is awake and breathing
Since pulmonary edema is a common spontaneously, is admitted for
complication after near drowning, the observation. Which action will be
nurse should assess the breath sounds most important for the nurse to take
frequently. The other information also will during the observation period?
be collected by the nurse, but it is not as
pertinent to the patient's admission
diagnosis.

vaccine. When planning the response to the


potential use of smallpox as an agent
rationale: of terrorism, the emergency
Smallpox infection can be prevented or department (ED) nurse-manager will
ameliorated by the administration of plan to obtain sufficient quantities of
vaccine given rapidly after exposure. The ______________
other interventions would be helpful for
other agents of terrorism but not for
smallpox.

The core temperature is 94° F (34.4° C). When rewarming a patient who
arrived in the emergency department
rationale: (ED) with a temperature of 87° F,
A core temperature of 89.6° F to 93.2° F which assessment indicates that the
(32° C to 34° C) indicates that sufficient nurse should discontinue the
rewarming has occurred. Dysrhythmias, rewarming?
hypotension, and shivering may occur
during rewarming and should be treated
but are not an indication to stop rewarming
the patient.

"Is someone at home hurting you?" When assessing a patient admitted to


the emergency department (ED) with
rationale: a broken arm and facial bruises, the
The nurse's initial response should be to nurse notes multiple additional
further assess the patient's situation. bruises in various stages of healing.
Telling the patient not to return home may Which statement or question by the
be an option once further assessment is nurse is most appropriate?
done. The patient, not the nurse, is
responsible for reporting the abuse. A
social worker may be appropriate once
further assessment is completed.

Give N-acetylcysteine (Mucomyst). A patient arrives in the emergency


department (ED) a few hours after
rationale: taking "20 to 30" acetaminophen
N-acetylcysteine is the recommended (Tylenol) tablets. Which action will
treatment to prevent liver damage after the nurse plan to take?
acetaminophen overdose. The other actions
might be used for other types of poisoning,
but they will not be appropriate for a
patient with acetaminophen poisoning.
assess the patient's current vital signs. A triage nurse in a busy emergency
department assesses a patient who
rationale: complains of 6/10 abdominal pain
The patient's pain and statement about an and states, "I had a temperature of
elevated temperature indicate that the 104.6º F (40.3º C) at home." The
nurse should obtain vital signs before nurse's first action should be to
deciding how rapidly the patient should be ______________
seen by the health care provider. A
urinalysis may be needed, but vital signs
will provide the nurse with the data needed
to determine this. The health care provider
will not order a medication before
assessing the patient.

A patient with a sucking chest wound The emergency department (ED)


triage nurse is assessing four victims
rationale: of an automobile accident. Which
Most immediate deaths from trauma occur patient has the highest priority for
because of problems with ventilation, so treatment?
the patient with a sucking chest wound
should be treated first. Face and head
fractures can obstruct the airway, but the
patient with facial injuries has lacerations
only. The other two patients also need
rapid intervention but do not have airway
or breathing problems.

Remove the patient's rings. The following actions are part of the
routine emergency department (ED)
rationale: protocol for a patient who has been
The patient's rings should be removed first admitted with multiple bee stings to
because it might not be possible to remove the hands. Which action should the
them if swelling develops. The other orders nurse take first?
also should be implemented as rapidly as
possible after the nurse has removed the
jewelry.

Assist with intubation of the patient. Gastric lavage and administration of


activated charcoal are prescribed for
rationale: an unconscious patient who has been
In an unresponsive patient, intubation is admitted to the emergency
done before gastric lavage and activated department (ED) after ingesting 30
charcoal administration to prevent diazepam (Valium) tablets. Which
aspiration. The other actions will be action will the nurse plan to take
implemented after intubation. first?

Place the patient in a shower. A patient arrives in the emergency


department after exposure to
rationale: radioactive dust. Which action should
The initial action should be to protect staff the nurse take first?
members and decrease the patient's
exposure to the radioactive agent by
decontamination. The other actions can be
done after the decontamination is
completed.

apply wet sheets and a fan to the patient. An unresponsive 78-year-old is


admitted to the emergency
rationale: department (ED) during a summer
The priority intervention is to cool the heat wave. The patient's core
patient. Antipyretics are not effective in temperature is 106.2° F (41.2° C),
decreasing temperature in heat stroke, and blood pressure (BP) 86/52, and pulse
100% oxygen should be given, which 102. The nurse initially will plan to
requires a high flow rate through a non- _____________
rebreather mask. An older patient would be
at risk for developing complications such
as pulmonary edema if given fluids at 1000
mL/hr.

Lung sounds When a patient is admitted to the


emergency department after a
rationale: submersion injury, which assessment
The priority assessment data are how well will the nurse obtain first?
the patient is oxygenating, so lung sounds
should be assessed first. The other data
also will be collected rapidly but are not as
essential as the lung sounds.

A patient with a red tag Following an earthquake, patients


are triaged by emergency medical
rationale: personnel and are transported to the
The red tag indicates a patient with a life- hospital. Which of these patients will
threatening injury requiring rapid the nurse need to assess first?
treatment. The other tags indicate patients
with less urgent injuries or those who are
likely to die.

Ask the family members about whether A patient's family members are in the
they would prefer to remain in the patient patient room when the patient has a
room or wait outside the room. cardiac arrest and emergency
personnel start resuscitation
rationale: measures. Which action is best for
Although many family members and the nurse to take initially?
patients report benefits from family
presence during resuscitation efforts, the
nurse's initial action should be to
determine the preference of these family
members. The other actions may be
appropriate, but this will depend on what is
learned when assessing family preferences.

c) A 22-year-old with multiple fractures of These four patients arrive in the


the face and jaw emergency department after a motor
a) A 72-year-old with palpitations and vehicle crash. In which order should
chest pain they be assessed?
b) A 45-year-old complaining of 6/10
abdominal pain
d) A 30-year-old with a misaligned right
leg with intact pulses

rationale:
The highest priority is to assess the 22-
year-old patient for airway obstruction,
which is the most life-threatening injury.
The 72-year-old patient may have chest
pain from cardiac ischemia and should be
assessed and have diagnostic testing for
this pain. The 45-year-old patient may
have abdominal trauma or bleeding and
should be seen next to assess circulatory
status. The 30-year-old appears to have a
possible fracture of the right leg and
should be seen soon, but this patient has
the least life-threatening injury.

You might also like